Question

Solve the 0-1 knapsack problem given the following items, each labeled with weight and value. Assume the total weight limit W

0 0
Add a comment Improve this question Transcribed image text
Answer #1

In the 0-1 Knapsack problem, we have to either include an item or not. We cannot include a part of an item, we either include it completely or not at all.

The way to solve this problem is by using the Dynamic programming approach. We see that each item can either be part of the optimal solution or not. So we first find the optimal solution for the same problem with size n-1. Then we add n to its solution and check whether it provides an optimal solution for the problem of size n, or not.

The answer for the above problem will be 94. By the sum of items of weights 6lbs and 2lbs.

item i 1 2 3 4
value val 8 40 30 54
weight wt 1 2 3 6
V[i,w] w = 0 1 2 3 4 5 6 7 8
i = 0 0 0 0 0 0 0 0 0 0
1 0 8 8 8 8 8 8 8 8
2 0 8 40 48 48 48 48 48 48
3 0 8 40 48 48 70 78 78 78
4 0 8 40 48 48 70 78 78 94

Hope this helps. If you have any queries or suggestions regarding the answers please leave them in the comments section so I can update and improve the answer. Thank you.

Add a comment
Know the answer?
Add Answer to:
Solve the 0-1 knapsack problem given the following items, each labeled with weight and value. Assume...
Your Answer:

Post as a guest

Your Name:

What's your source?

Earn Coins

Coins can be redeemed for fabulous gifts.

Not the answer you're looking for? Ask your own homework help question. Our experts will answer your question WITHIN MINUTES for Free.
Similar Homework Help Questions
  • In weighted knapsack problem, given the knapsack capacity is 16 and the following items (Weight, Value),...

    In weighted knapsack problem, given the knapsack capacity is 16 and the following items (Weight, Value), what is the maximum value we can take away. Explain shortly how and by what approach you arrived at this solution. Item 1 (4, 12) Item 2 (3, 14) Item 3 (7, 22) Item 4 (8, 32) Item 5 (4, 24) Item 6 (6, 20)

  • In Python, write the following function: Knapsack Problem: Given a set of items, each with a...

    In Python, write the following function: Knapsack Problem: Given a set of items, each with a weight and a value, determine the number of each item to include in a collection so that the total weight is less than or equal to a given limit and the total value is maximized.

  • Please explain step by step, thank you so much! 0-1 Knapsack (N=6, W=10) Item Weight Value...

    Please explain step by step, thank you so much! 0-1 Knapsack (N=6, W=10) Item Weight Value (lb) ($) 8 1 0 10 Weight limit w(lb) 4 5 6 7 2 2 2 2 2 2 3 2 #2 2 1 2 43 33 3 w #3 0 2 3 3 #4 56 w 2 a #6 7. (10%) (Cont.) Unbounded Knapsack Problem (1-Dim Dynamic Programming) Weight limit w 0 : 6 Weight limit w F(w) 7 Unbounded Knapsack (N=6, W=10)...

  • 5) (10 pts) Greedy Algorithms The 0-1 Knapsack problem is as follows: you are given a...

    5) (10 pts) Greedy Algorithms The 0-1 Knapsack problem is as follows: you are given a list of items, each item has an integer weight and integer value. The goal of the problem is to choose a subset of the items which have a sum of weights less than or equal to a given W with a maximal sum of values. For example, if we had the following five items (each in the form (weight, value)): 11(6, 13), 2(4, 10),...

  • "Greedy, but Better": Given a knapsack problem with a weight capacity C, and n items, and...

    "Greedy, but Better": Given a knapsack problem with a weight capacity C, and n items, and each item has a weight W[1:n] and monetary value P[1:n]. You have to determine which items to take so that the total weight is C, and the total value (profit) is maximized. In this case we are considering an integer problem, so you can either take an item, or not take an item, you cannot take it fractionally. If you recall, the greedy algorithm...

  • 1. Apply the dynamic programming algorithm discussed in class to solve the knapsack problem. (10 points)...

    1. Apply the dynamic programming algorithm discussed in class to solve the knapsack problem. (10 points) a. Show the completed table. b. Which items are included in the final configuration of the knapsack? c. What is the maximum value that can fit in the knapsack using a configuration of these items? item 1 2. 3 4 weight 3 2 value $25 $20 $15 1 capacity W = 6. 4 5 $40 $50 5

  • Recall that in the "Knapsack Problem", there are n items having respective values V1..n) and weights...

    Recall that in the "Knapsack Problem", there are n items having respective values V1..n) and weights W1..n), all greater than 0 and one needs to maximize the total value of the subset of the items placed in the knapsack limited by a weight capacity of W In the 0-1 Knapsack Problem, each item must be either be included or excluded in its entirety, in light of the fact that this problem is to be "NP-Complete", how can one solve the...

  • 2 Knapsack Problem In a Knapsack problem, given n items {11, I2, -.., In} with weight {wi, w2, -.., wn) and value fvi,...

    2 Knapsack Problem In a Knapsack problem, given n items {11, I2, -.., In} with weight {wi, w2, -.., wn) and value fvi, v2, ..., vn], the goal is to select a combination of items such that the total value V is maximized and the total weight is less or equal to a given capacity W. Tt i=1 In this question, we will consider two different ways to represent a solution to the Knapsack problem using an array with size...

  • Local Search Knapsack 0-1

    Design a local search algorithm for the 0-1 knapsack problem. Assume there are n items x1 ... xn each with weight wi and value vi. The knapsack can have at most one of each item and the total weight cannot exceed W. You want to maximize the total value in the knapsack.Question 1: (7 points) Show the psuedocode/explanation for your algorithm.Question 2. (3 points) Is it guaranteed to find an optimal solution? Justify your answer.

  • The decision version of the Knapsack problem is as follows: Given a set of n items...

    The decision version of the Knapsack problem is as follows: Given a set of n items {1, 2, …, n}, where each item j has a value v(j) and a weight w(j), and two numbers V and W, can we find a subset X of {1, 2, …, n} such that Σj∈X v(j) ≥ V and Σj∈X w(j) ≤ W? Prove formally that the Knapsack problem is NP-complete.

ADVERTISEMENT
Free Homework Help App
Download From Google Play
Scan Your Homework
to Get Instant Free Answers
Need Online Homework Help?
Ask a Question
Get Answers For Free
Most questions answered within 3 hours.
ADVERTISEMENT
ADVERTISEMENT
ADVERTISEMENT